Line 1: Line 1:
 +
[[Category:MA453Spring2009Walther]]
 +
 
Are the following irreducible over Q?
 
Are the following irreducible over Q?
  

Revision as of 18:14, 8 April 2009


Are the following irreducible over Q?

a) $ x^5 + 9x^4 + 12x^2 + 6 $ b) $ x^4 + x + 1 $ c) $ x^4 + 3x^2 + 3 $ d) $ x^5 + 5x^2 + 1 $ e) $ (5/2)x^5 + (9/2)x^4 + 15x^3 + (3/7)x^2 + 6x + (3/14) $



a.) Look at Eisenstein's with p = 3.
b.) A polynomial is irreducible in Q if there's a p such that f(x) mod p is irreducible. Look at p = 2.
c.) See part a.
d.) See part b.
e.) Multiply by 14 then see part a.
--Jniederh 22:12, 8 April 2009 (UTC)

Alumni Liaison

To all math majors: "Mathematics is a wonderfully rich subject."

Dr. Paul Garrett